在与拆分一起使用子方程时,难以正确编号方程

在与拆分一起使用子方程时,难以正确编号方程

不明白为什么方程的最后一行会多出一个方程号

 \begin{subequations}
    \label{eq: non dimensional lagrangian}
     \begin{align}
     \mathcal{L}&=\mathcal{T}-\mathcal{V} \nonumber \\
       \mathcal{V}&=\frac{EI_{zz}}{2L}\, \int_{0}^{1} \hspace{-0.05in} \left(\frac{\partial^2\psi(\eta)}{\partial \eta^2}\right)^2 \mathrm{d}\eta +\frac{L^2}{2}\sum_{j=1}^{n}\,k_{1,j}\left(\psi(\eta)\right)^2 \nonumber\\
        \mathcal{T}&=\frac{\rho A\omega^2L^3}{2}\, \int_{0}^{1}  \hspace{-0.05in} \left( \psi(\eta)\right)^2 \mathrm{d}\eta \nonumber \\
        \begin{split}
     \mathcal{\bar{L}}&=\frac{\beta^4}{2}\, \int_{0}^{1}  \hspace{-0.05in} \left( \psi(\eta)\right)^2 \mathrm{d}\eta-\frac{1}{2}\, \int_{0}^{1} \hspace{-0.05in} \left(\frac{\partial^2\psi(\eta)}{\partial \eta^2}\right)^2 \mathrm{d}\eta\\ &\quad{}-\frac{1}{2}\sum_{j=1}^{n}\,K_{1,j}\left(\psi(\eta)\right)^2    
     \end{split}\\ \nonumber \\
     \end{align}
     \end{subequations}

答案1

\\您的输入有错误。

如果您希望数字位于最后一行旁边,请不要使用split

\documentclass[twocolumn]{article}
\usepackage{amsmath}

\newcommand{\diff}{\mathop{}\!\mathrm{d}}

\begin{document}

\begin{subequations}\label{eq: non dimensional lagrangian}
\begin{align}
\mathcal{L} &= \mathcal{T}-\mathcal{V} \nonumber \\
\mathcal{V} &= \frac{EI_{zz}}{2L} \int_{0}^{1} \left(
                 \frac{\partial^2\psi(\eta)}{\partial \eta^2}
               \right)^2 \diff\eta +
               \frac{L^2}{2}\sum_{j=1}^{n} k_{1,j}(\psi(\eta))^2 \nonumber \\
\mathcal{T} &= \frac{\rho A\omega^2L^3}{2} \int_{0}^{1} (\psi(\eta))^2 \diff\eta \nonumber \\
\bar{\mathcal{L}} &= \frac{\beta^4}{2} \int_{0}^{1} (\psi(\eta))^2 \diff\eta -
                     \frac{1}{2} \int_{0}^{1} \left(
                       \frac{\partial^2\psi(\eta)}{\partial \eta^2}
                     \right)^2 \diff\eta \nonumber \\
                  &\quad-\frac{1}{2}\sum_{j=1}^{n}K_{1,j}(\psi(\eta))^2    
\end{align}
\end{subequations}

\end{document}

我对您的输入做了一些修改,特别是删除了无用的\left\right。此外\mathcal{\bar{L}},虽然 产生了预期的结果,但 应该是 ,\bar{\mathcal{L}}这在逻辑上更为合理。

使用\diff而不是显式\mathrm{d}(有很多理由)。我删除了您使用的显式间距。

在此处输入图片描述

您可能会觉得split更合乎逻辑,我同意。在这种情况下,tbtags在呼叫时添加选项amsmath,所有数字将与拆分中的底线对齐。

相关内容